NBME Explanations

New Launch of Test Pirates Downloads with NBME answers and explanations

NBME Explanations

NBME 16 Questions and Answers

Discussion in 'USMLE Step 1' started by orthopod, May 24, 2015.

  1. ymma

    New Member
    Expand Collapse

    Joined:
    Jun 27, 2016
    Messages:
    2
    Likes Received:
    0
    Gender:
    Female
    Location:
    caribe
    13. Experiment, the concentration of glycolic intermediates is measured in human endothelial cells incubated with 30 mM glucose compared with 5 mM. Results shown. Activity of which enzymes is most likely inhibited indirectly by increased glucose concentrations?
    - Hexokinase

    is incorrect, I put this one and got it wrong.

    Anyone know the real answer?
     
  2. usmle 2016

    New Member
    Expand Collapse

    Joined:
    Aug 8, 2016
    Messages:
    1
    Likes Received:
    0
    Gender:
    Male
    Location:
    pakistan
    block 2 why in 49 all increased?
     
  3. Mohammad Taha

    New Member
    Expand Collapse

    Joined:
    Jun 15, 2016
    Messages:
    12
    Likes Received:
    3
    Gender:
    Male
    Location:
    241 Riley Ave. Macon, GA 31204 USA
    #23 Mohammad Taha, Aug 17, 2016
    Last edited: Aug 17, 2016
    Block 2/ question 13
    I chose the same answer A) abnormal origin of superior mesenteric artery, and I got that wrong.
    In patients with horseshoe kidney, the central portion of the kidney may be found just inferior to the inferior mesenteric artery (not SMA) because the normal embryologic ascent of the kidneys is arrested by its presence in people with central fusion of the kidneys.
    I think right answer is B) anomalous origins of multiple multiple renal arteries to each kidney!
     
    orthopod and awesomefo like this.
  4. Mohammad Taha

    New Member
    Expand Collapse

    Joined:
    Jun 15, 2016
    Messages:
    12
    Likes Received:
    3
    Gender:
    Male
    Location:
    241 Riley Ave. Macon, GA 31204 USA
    Block 4/ question 22
    I chose A) autosomal dominant and I got it wrong.
    Now I think the right answer is B) autosomal recessive inheritance!
     
  5. awesomefo

    New Member
    Expand Collapse

    Joined:
    Sep 19, 2016
    Messages:
    5
    Likes Received:
    2
    Gender:
    Female
    Location:
    New Jersey
    18. Study to assess 32 patients in a community of 1000 who have developed drug-resistant tuberculosis during a 1-year period. Patients removed from the community for Rx. Assuming risk for infection and susceptibility to disease is constant, which best represents the number most likely to develop drug-resistant TB during the next year?
    - 32


    ^^ for this one, I had put 31 (calculated it and got 30.976), and the expanded feedback didn't say I got it wrong. let me know if you guys agree
     
    Dystopian and ecraft1592 like this.
  6. awesomefo

    New Member
    Expand Collapse

    Joined:
    Sep 19, 2016
    Messages:
    5
    Likes Received:
    2
    Gender:
    Female
    Location:
    New Jersey
    41. 20-year-old F with BP 140/100. BUN 50, glucose 90. U/A shows numerous WBCs and WBC casts; protein excretion is 3000 mg/24h. GFR is 20% of normal. Ultrasonography shows small asymmetric kidneys with broad scars and blunted calyces, with vesicoureteral reflux. Dx?
    - Bilateral hydronephrosis

    ^^ I actually believe I put Chronic pyelonephritis for this one and it was not marked wrong. Let me know if you agree with me on this
     
  7. awesomefo

    New Member
    Expand Collapse

    Joined:
    Sep 19, 2016
    Messages:
    5
    Likes Received:
    2
    Gender:
    Female
    Location:
    New Jersey
    32.16yo boy 30 minutes after he dove into a 3-foot-deep pool at night. Unable to move right upper/lower extremities. Damage to which labeled region of cross section of spinal cord?
    - G

    This answer is E, I also put G but the expanded feedback highlighted E
     
  8. awesomefo

    New Member
    Expand Collapse

    Joined:
    Sep 19, 2016
    Messages:
    5
    Likes Received:
    2
    Gender:
    Female
    Location:
    New Jersey
    #28 awesomefo, Sep 22, 2016
    Last edited: Sep 22, 2016
    A 38 YO female undergoes laparoscopic cholecystectomy w general anesthesia. After she wakens postoperatively, she is nauseated and vomits three times in 20 mins. Which of the following is the most appropriate treatment for the nausea and vomiting in this patient?

    D Odansetron, confirmed by feedback
     
  9. awesomefo

    New Member
    Expand Collapse

    Joined:
    Sep 19, 2016
    Messages:
    5
    Likes Received:
    2
    Gender:
    Female
    Location:
    New Jersey
    3. 43-year-old F with difficulty walking for 3 months. Weaknessand decreased muscle bulk of the lower extremities. Patellar and Achilles tendon reflex diminished. Pain/temp, position normal. Cause?

    B. Degeneration of motoneurons of the Lumbar cord, I don't know why, it was on expanded feedback.
     
  10. smile

    New Member
    Expand Collapse

    Joined:
    Sep 26, 2016
    Messages:
    7
    Likes Received:
    2
    Gender:
    Male
    Location:
    south korea

    The ans is pyrentel and the organism is schistosoma. Mansoni / japonicum.
     
  11. smile

    New Member
    Expand Collapse

    Joined:
    Sep 26, 2016
    Messages:
    7
    Likes Received:
    2
    Gender:
    Male
    Location:
    south korea
    Sincere Thanks to orthopod for all your efforts in putting up q. And ans.
     
    orthopod likes this.
  12. smile

    New Member
    Expand Collapse

    Joined:
    Sep 26, 2016
    Messages:
    7
    Likes Received:
    2
    Gender:
    Male
    Location:
    south korea
    I p
    I put autosomal recessive and got it wrong.
     
  13. ajend

    New Member
    Expand Collapse

    Joined:
    May 2, 2016
    Messages:
    5
    Likes Received:
    0
    Gender:
    Male
    Occupation:
    medical student
    Location:
    chicago
    #33 ajend, Jan 10, 2017
    Last edited: Jan 10, 2017
    Block 4 part 1
    question 3. ans= demyelination of the cortical pathway is not correct (i got that wrong). i am thinking about B. (degeneration of motoneurons of the lumber cord.) is the right answer. but not sure!
     
  14. ajend

    New Member
    Expand Collapse

    Joined:
    May 2, 2016
    Messages:
    5
    Likes Received:
    0
    Gender:
    Male
    Occupation:
    medical student
    Location:
    chicago
    PFK is wrong too
     
  15. ajend

    New Member
    Expand Collapse

    Joined:
    May 2, 2016
    Messages:
    5
    Likes Received:
    0
    Gender:
    Male
    Occupation:
    medical student
    Location:
    chicago
    I got low specificity wrong in my feedback
     
  16. ajend

    New Member
    Expand Collapse

    Joined:
    May 2, 2016
    Messages:
    5
    Likes Received:
    0
    Gender:
    Male
    Occupation:
    medical student
    Location:
    chicago
    Block 2 part 1 #18. I got my answer of 32 wrong in my feedback. 31 is the correct ans.
     
  17. ajend

    New Member
    Expand Collapse

    Joined:
    May 2, 2016
    Messages:
    5
    Likes Received:
    0
    Gender:
    Male
    Occupation:
    medical student
    Location:
    chicago
    31 is write. According to my feedback
     
  18. shiva

    New Member
    Expand Collapse

    Joined:
    Jan 11, 2017
    Messages:
    9
    Likes Received:
    2
    Gender:
    Male
    Location:
    india
    ilioinguinal nerve q is wrong ...answer was pudendal nerve..if possible please change
     
  19. shiva

    New Member
    Expand Collapse

    Joined:
    Jan 11, 2017
    Messages:
    9
    Likes Received:
    2
    Gender:
    Male
    Location:
    india
    can anyone explain why?
     
  20. shiva

    New Member
    Expand Collapse

    Joined:
    Jan 11, 2017
    Messages:
    9
    Likes Received:
    2
    Gender:
    Male
    Location:
    india
    answer is wrong
    anomalous origin of multiple renal arteries
     
    orthopod likes this.

Share This Page